1answer.
Ask question
Login Signup
Ask question
All categories
  • English
  • Mathematics
  • Social Studies
  • Business
  • History
  • Health
  • Geography
  • Biology
  • Physics
  • Chemistry
  • Computers and Technology
  • Arts
  • World Languages
  • Spanish
  • French
  • German
  • Advanced Placement (AP)
  • SAT
  • Medicine
  • Law
  • Engineering
LenaWriter [7]
2 years ago
7

Use the diagram below to find the indicated ratios.

Mathematics
1 answer:
Tanzania [10]2 years ago
3 0

Answer:

Sin A = 20/29

Cos A = 21/29

Tan A = 20/21

Sin C = 21/29

Cos C = 20/29

Tan C = 21/20

Step-by-step explanation:

Sine is opp/hyp, cosine is adj/hyp, and tangent is opp/adj.

You might be interested in
What are the total calories in a sandwich if it contains 25 grams of total fat?
posledela

Answer:

i think 480 total calories

Step-by-step explanation:

i wish it's true

8 0
2 years ago
Hello, Brainly community!
ioda

Answer:

(B)  \displaystyle \frac{W(3.1) - W(2.9)}{0.2}

General Formulas and Concepts:

<u>Calculus</u>

Limits

Derivatives

  • The definition of a derivative is the slope of the tangent line.

Derivative Notation

Instantaneous Rates

  • Tangent Line: \displaystyle f'(x) = \frac{f(b) - f(a)}{b - a}

Step-by-step explanation:

Since we are trying to find a <em>rate</em> at which W(t) changes, we must find the <em>derivative</em> at <em>t</em> = 3.

We are given 2 close answer choices that would have the same <em>numerical</em> answer but different <em>meanings</em>:

  1. (A)  \displaystyle  \lim_{t \to 3} W(t)
  2. (B)  \displaystyle \frac{W(3.1) - W(2.9)}{0.2}

If we look at answer choice (A), we see that our units would simply just be volume. It would not have the units of a rate of change. Yes, it may be the closest numerically correct answer, but it does not tell us the <em>rate</em> at which the volume would be changing and it is not a derivative.

If we look at answer choice (B), we see that our units would be cm³/s, and that is most certainly a rate of change. Answer choice (B) is also a <em>derivative</em> at <em>t</em> = 3, and a derivative tells us what <em>rate</em> something is changing.

∴ Answer choice (B) will give us the best estimate for the value of the instantaneous rate of change of W(t) when <em>t</em> = 3.

Topic: AP Calculus AB/BC (Calculus I/I + II)

Unit: Differentiation

Book: College Calculus 10e

8 0
2 years ago
X=y-7. <br> X+8y=2<br> Hellplpp helllpppp! Plz!
Luden [163]
X= -5/7
y= 44/7

Hope this helps!!

6 0
3 years ago
Please help like right now ABC = RSO<br> Find m
Alja [10]

Answer:

50⁰

Step-by-step explanation:

the 2 triangles are congruent, meaning the angles equal the same. angles B and S are the same, angles A and R are the same, and angles Q and C are the same (congruent)

6 0
3 years ago
What is the answer to the qustion 0.3h+1.2=0.1h-2.6
andreev551 [17]

Answer:

-19

Step-by-step explanation:

0.3h+1.2=0.1h-2.6

.2h+1.2=-2.6

.2h=-3.8

h=-19

7 0
3 years ago
Other questions:
  • -29/35 + 17/25 give your answer as a reduced fraction
    14·1 answer
  • In 1999 bus fare in atlanta was 4.70.In 1979 the fare was 2/5 of the fare in 1999 what was the fare in 1979
    5·2 answers
  • Do you times two by the radius to equal diameter?​ I need to make sure.
    6·1 answer
  • What is the midpoint of a segment shown below?
    6·2 answers
  • Find the rule to complete the pyramid.
    8·1 answer
  • In a 135-pound adult, about 94 pounds of the adult’s weight is water. What percent of the weight is water, rounded to the neares
    5·1 answer
  • HELP MEEEEEEEEEEEEEEEEEEEEEEEEEEEEEEEEEEEEE
    14·1 answer
  • Median weekly earnings for women with some college for an associate degree.
    13·1 answer
  • (iv) In the figure, there are(a) one ray and an intersecting point (b) two rays and an intersecting point(c) one ray and two int
    8·1 answer
  • prove “if two angles of one triangle are congruent to two angles of a second triangle, the the third angles of the triangles are
    14·1 answer
Add answer
Login
Not registered? Fast signup
Signup
Login Signup
Ask question!